EENT - Hippo study questions

Ace your homework & exams now with Quizwiz!

A 2-year-old boy is brought to the office by his father for evaluation of painful mouth sores for the past two days. Four days ago, the child began having general malaise, anorexia, and sleeplessness. He also developed a fever with a temperature of up to 39.0ºC (102.2ºF). Pain associated with the mouth lesions prevents the boy from wanting to eat or drink. Physical examination of the oropharynx shows several clusters of small vesicles with erythematous bases and areas of ulceration, both of which bleed easily, on the lips, buccal mucosa, tongue, gingiva, and hard palate. Which of the following would be seen on a Tzanck smear from the base of one of his lesions?

1) A fern pattern 2) Atypical lymphocytes 3) Clue cells 4) Motile organisms 5) Multinucleated giant cells Multinucleated giant cells on Tzanck smear is indicative of herpes simplex infection. Oral herpes simplex presents with multiple painful, grouped vesicular lesions. Patients may also have a prodrome of pain, tingling, or burning. The multinucleated giant cells seen on the Tzanck smear can be seen in VZV as well as HSV so more sensitive and specific tests are used for definitive diagnosis such as viral polymerase chain reaction (PCR) or direct fluorescent antibody (DFA) assay.

A 9-year-old girl is brought to the clinic because she has had right eye pain for the past three days. She has had a runny nose and pressure in the front of her face for the past week. Temperature is 39.0 C (102.2 F). On physical examination, the frontal and maxillary sinuses are tender to palpation. The right eyelid is erythematous and edematous. The right orbital area is exquisitely tender to palpation. Chemosis is noted. Based on these findings, which of the following additional physical examination findings is most likely to be present?

1) A small painful papule at the eyelid margin - A small painful papule at the eyelid margin would suggest a hordeolum. A hordeolum may cause very mild eyelid erythema and swelling but is not associated with exquisite tenderness to the entire orbital area, chemosis, or fever. A hordeolum is typically managed with warm compresses and symptomatic relief. 2) Normal results on vision examination - Because orbital cellulitis affects the extraocular muscles and the soft tissues in the orbit, it can cause diplopia and ophthalmoplegia. Orbital cellulitis can progress to an abscess and cause vision loss or even death. A decreased pupillary light reflex or an afferent pupillary light defect would suggest the optic nerve is involved. 3) Pain relief with eye movements - The most likely diagnosis is orbital cellulitis, which is characterized by eyelid swelling with erythema, eye pain, and pain with eye movements. Chemosis and fever are more common in orbital cellulitis than in preseptal cellulitis. This is because orbital cellulitis results in inflammation and swelling of the extraocular muscles and tissues in the orbit itself, which causes pain with eye movements and proptosis. 4) Proptosis - Proptosis is a physical exam finding likely to be present. Orbital cellulitis is most commonly caused by rhinosinusitis and is characterized by eyelid swelling with erythema, eye pain, and pain with eye movements. Chemosis and fever are more common in orbital cellulitis than in preseptal cellulitis. 5) Scattered erythematous vesicles unilaterally - Scattered erythematous vesicles unilaterally at the orbital area would suggest herpes zoster ophthalmicus. Although it can be sight threatening, the usual symptoms would be a prodrome of headache and malaise. A painful "burning" vesicular rash is commonly reported.

A 74-year-old female presents for a routine checkup. She reports that over the past year, her vision is gradually worsening. She also admits that colors seem less vibrant and that glare bothers her, especially at night. Which of the following would you expect to see on physical exam?

1) A vascular triangular mass on the nasal side of the cornea 2) Afferent pupillary defect 3) Circumlimbal injection and a steamy cornea with a fixed mid-dilated pupil 4) Opalescent changes to the lens - This is the physical exam finding that you would expect with cataracts. You may also notice cataracts on fundoscopic exam as black areas on a red background. 5) Subretinal neovascularization

A 36-year-old man comes to the clinic because he has had persistent ringing in the ears for the past three months. He describes the sound as pulsatile and bilateral, and it increases in frequency and intensity with exercise. He has no current medical conditions and takes no medications. During the interview, he says that he was involved in a motor vehicle collision five years ago in which he sustained a head injury. Physical examination shows no evidence of overt auditory disease. However, slight-to-moderate high-frequency sensory hearing loss is noted in both ears. Decreased sensation also is noted in the left upper extremity. Based on this presentation, this patient is at increased risk for which of the following?

1) Aberrant carotid artery tumor - Aberrant carotid artery tumor associated tinnitus presents with a machinelike grinding or pulsing character and is also sometimes associated with intracranial lesions, such as chondrosarcoma and endolymphatic sac tumors. 2) Dural arteriovenous fistula - In a study of patients with pulsatile tinnitus seen in a neurology department, a vast majority were found to have a significant vascular disorder (most commonly dural arteriovenous fistula). Dural arteriovenous fistulas are often associated with dural venous sinus thrombosis, which may occur spontaneously or be associated with infection, tumor, trauma, or surgery. Large dural AVFs can result in intracranial hemorrhage, and thus early detection and treatment (surgery and/or vascular embolization) can be lifesaving. Symptoms can include both hearing issues (pulsatile tinnitus) and vision problems including visual deterioration, eye bulge, swelling in the eye lining, eye-related palsies, and cavernous sinus syndrome. 3) Eustachian tube dysfunction - A patulous eustachian tube can cause tinnitus with sounds similar to an ocean roar that may be synchronous with respiration. It most commonly occurs after significant weight loss or after external beam radiation to or near the nasopharynx. The symptoms may disappear when the patient lies down. 4) Multiple sclerosis - Auditory symptoms of multiple sclerosis include clicking noises or irregular or rapid pulsations in the ears which may result from myoclonus of the palatal muscles that attach to the Eustachian tube orifice. Myoclonus of the palatal muscles most often is caused by an underlying neurologic abnormality. 5) Temporomandibular joint dysfunction - Temporomandibular joint (TMJ) dysfunction tinnitus is typically nonpulsatile and associated with whiplash injuries and other disorde

Which of the following physical examination characteristics best differentiates oral candidiasis from oral leukoplakia?

1) Ability to scrape off patch - Oral candidiasis, or thrush, is characterized by white plaques on the tongue, buccal mucosa, or palate, that with effort can be scraped away. There is commonly underlying erythema. Remember, 'thrush will brush'. It occurs in immunocompromised patients, patients using corticosteroid inhalers, patients who use dentures, and in those recently treated with antibiotics. Oral leukoplakia presents with white plaques of the oral mucosa. However, these can not be scraped away. Leukoplakia is a precancerous lesion. Risk factors include smoking and chewing tobacco. 2) Color of patch 3) Induration of patch 4) Location of patch 5) Particular shade or hue of white

A 22-year-old female college student comes to the urgent care clinic because she has had a sore throat and fever to 38.3º C (101.0º F) for the past two days. She says her sore throat pain is sharp and occurs with swallowing but says that she is able to tolerate oral liquids and foods. The patient also has had a slight headache and fatigue; she has not had neck stiffness, nasal congestion, rhinorrhea, sneezing, ear pain, or conjunctivitis. She admits that a few people living in her college dormitory had similar symptoms last week. Vital signs are within normal limits except for a temperature of 38.9º C (102.0º F). Physical examination shows an erythematous posterior pharynx and edematous tonsils with white tonsillar exudates. According to the Centor criteria, which of the following additional physical examination findings is most likely to indicate the need for a rapid strep test?

1) Absence of cough - This clinical vignette describes a patient with acute bacterial pharyngitis given rapid onset of sore throat, fever, lymphadenopathy, and tonsillar exudates. Centor criteria help in the medical decision making process when evaluating for Group A Streptococcus (GAS). Centor criteria include: tonsillar exudates, tender anterior cervical lymphadenopathy, fever, and absence of cough. Each criterion represents one point, and the likelihood of GAS increases as total points rise. Typically, clinicians may decide to order a rapid strep test when there are at least two to three positive Centor criteria. 2) Cobblestoning of the posterior pharynx - This is a finding most consistent with allergic rhinitis. The patient in the clinical vignette denies nasal congestion and rhinorrhea. 3) Presence of drooling - Drooling is commonly a symptom of upper airway obstruction and not a part of Centor criteria. In this clinical vignette, the patient does not demonstrate any "red flag" symptoms to indicate upper airway obstruction. 4) Presence of rash - Although a scarlatiniform rash can be seen in GAS, it is not part of the Centor criteria. 5) Wheezing on auscultation of the lungs - It is not typical of patients with GAS to present with pulmonary symptoms or findings, nor is wheezing part of Centor criteria.

A 52-year-old woman comes to the emergency department because she has had severe pain in the right eye, decreased visual acuity, and diminished peripheral vision with associated nausea for the past 30 minutes. She has not had any recent trauma, and medical history is unremarkable. On physical examination, the right pupil appears mid-dilated. Tonometry shows increased intraocular pressure. Which of the following is the most appropriate initial treatment?

1) Acetazolamide This is the most appropriate first-line treatment for acute angle-closure glaucoma and is typically given intravenously. Acetazolamide's mechanism of action involves the inactivation of carbonic anhydrase, therefore decreasing intraocular pressure by decreasing the aqueous humor production. 2) Glycopyrrolate 3) Pilocarpine 4) Timolol 5) Travoprost

A 34-year-old transgender woman comes to the urgent care clinic because of nausea, vomiting, vertigo, and some hearing loss in both ears for the past day. She states that three days prior, she had a runny nose, mild sore throat, and some plugging of both ears. Which of the following best explains this patient's symptoms?

1) Acoustic neuroma - Acoustic neuroma results in unilateral sensorineural hearing loss. 2) Eighth cranial nerve palsy - Eighth cranial nerve palsy would result in unilateral sensorineural hearing loss but would be associated with disequilibrium. 3) Labyrinthitis - Labyrinthitis is an inflammation of the inner ear that results in severe vertigo, tinnitus, and possibly hearing loss, nausea and vomiting. It can occur as a single attack, a series of attacks, or a persistent condition that usually diminishes over time. Vestibular neuronitis may also be associated with nystagmus. The etiology is not always clear, but it may be associated with infections (viral or bacterial), head injury, or allergy. 4) Meniere disease - Meniere disease is best characterized by a pathophysiology involving distention of the endolymphatic compartment leading to chronic episodic vertigo, progressive low-frequency sensorineural hearing loss, and tinnitus. 5) Otosclerosis - Progressive abnormal remodeling of the bones of the inner ear resulting in unilateral (if one-sided) or bilateral (if both sides) conductive hearing loss.

A 68-year-old woman comes to the primary care office because she has had intermittent episodes of dizziness that last between five to 10 minutes during the past several weeks. She says the episodes are triggered by quick head motions. The patient has not had nausea and vomiting, hearing changes, or gait instability. During the physical examination, the Dix-Hallpike maneuver is performed and transient upbeat-torsional nystagmus is noted. Which of the following is the most likely the cause of her vertigo?

1) Acoustic neuroma - An acoustic neuroma is a benign tumor that grows on the nerve that innervates the inner ear. Symptoms typically include hearing impairment, unsteadiness, and tinnitus. 2) Benign paroxysmal positional vertigo - BPPV is the most common cause of vertigo in the elderly caused by calcium crystals being dislodged in the inner ear. 3) Meniere's disease - Meniere's disease is characterized by a triad of hearing loss, dizziness, and tinnitus. 4) Orthostatic hypotension - Orthostatic hypotension is described as a drop in blood pressure that may lead to dizziness. 5) Vestibular neuronitis - Vestibular neuronitis generally has a viral cause. It is characterized by inflammation of the vestibular nerve causing nausea, vomiting and dizziness.

A 46-year-old male presents with acute onset of a bright red blood patch on the lateral portion of his right sclera. He denies pain or light sensitivity. Visual acuity testing reveals no defect on the right side. Vital signs are within normal limits. The only other symptom that the patient is experiencing at this time is a cough, which has been improving over the last few days. Which of the following is the most appropriate next step in the management of this patient? 1) Admission to the hospital with IV acetazolamide administration 2) CT scan of the head 3) Emergency ophthalmology consult 4) No treatment is necessary 5) Order a complete blood count and bleeding time

1) Admission to the hospital with IV acetazolamide administration 2) CT scan of the head 3) Emergency ophthalmology consult 4) No treatment is necessary 5) Order a complete blood count and bleeding time

A 32-year-old man, who deep water dives for recreation, comes to your clinic complaining of bilateral ear pain and tinnitus for the last three days. His symptoms began after completing a dive with a friend. He denies ever having these symptoms before but admits that they did have to cut their dive short and ascended quicker than usual. Which of the following is the most likely diagnosis?

1) Acoustic neuroma - An acoustic neuroma is a tumor of the vestibulocochlear nerve. It results in unilateral sensorineural hearing loss and continuous vertigo. Tinnitus may also be present. It can be diagnosed with an MRI. 2) Acute otitis media - Otitis media involves eustachian tube dysfunction, leading to a bacterial infection of the ear. Patients will have a red, bulging tympanic membrane, with limited mobility. In mild disease, conservative management is reasonable. In moderate to severe disease, oral antibiotics are used for treatment. 3) Auditory tube dysfunction - Eustachian tube dysfunction can involve impaired pressure regulation, poor fluid clearance, or impaired ventilation of the middle ear. Patients may complain of hearing loss and have chronic middle ear effusion. Short term use of nasal decongestants can be helpful to reduce ear pressure. If symptoms are severe or persistent, surgery may be required. 4) Barotrauma - Given this clinical scenario, barotrauma is the most likely cause of this man's symptoms. If tinnitus or vertigo is present, it may indicate trauma to the inner ear. These patients can be counseled to swallow or yawn to attempt to auto-inflate the Eustachian tube. Most injuries heal on their own, but decongestants or myringotomy may also be indicated. 5) Swimmer's ear - Though a part of the differential diagnosis, swimmer's ear, or otitis externa, is not as likely a diagnosis as barotrauma. Water exposure to the external auditory canal can lead to infection or inflammation. Patients may have pruritus, pain, canal edema, and purulent ear discharge. Treatment involves topical antibiotics, steroids, and ear cleaning.

A 67-year-old woman, who has been recovering from an upper respiratory tract infection, comes to the clinic today complaining of acutely feeling "the room spinning". She admits having trouble walking at times and has stumbled, but not fallen yet. She denies having hearing loss. On physical examination, you note a horizontal nystagmus with moderate amplitude and frequency. Which of the following is the most likely diagnosis?

1) Acoustic neuroma 2) Benign paroxysmal positional vertigo 3) Cavernous sinus thrombosis 4) Meniere's disease 5) Vestibular neuronitis Vertigo that presents after an upper respiratory illness is either vestibular neuronitis and/or labyrinthitis. Pure vestibular neuronitis is distinguished from labyrinthitis in that vestibular neuronitis is acute vertigo with preserved auditory function, while labyrinthitis is acute vertigo with loss of hearing. Vestibular neuronitis can be treated with steroids and antiemetics.

Which of the following diseases is best characterized by a pathophysiology involving distention of the endolymphatic compartment leading to chronic episodic vertigo, progressive low-frequency sensorineural hearing loss, and tinnitus?

1) Acoustic neuroma 2) Cholesteatoma 3) Chronic otitis media 4) Labyrinthitis 5) Meniere's disease - While the exact pathophysiology of Meniere disease is controversial, it is thought that the underlying mechanism is due to distortion of the membranous labyrinth resulting from overaccumulation of endolymph. Changes in pressure within this compartment stresses nerve-rich membranes which causes hearing disturbances, tinnitus, vertigo, imbalance, and a pressure sensation within the ear.

A 6-year-old girl is brought to the pediatric otolaryngology clinic by her mother because she has had intermittent fluid drainage from her left ear over the past 18 months. The patient has been evaluated at the clinic several times in the past due to recurrent otitis media, and medical history includes placement of tympanostomy tubes from the ages of 1 to 3. Physical examination shows a pearly white mass behind the left tympanic membrane. Current audiometric testing shows a conductive hearing loss in the left ear. Which of the following underlying conditions accounts for the pathophysiology of this patient's diagnosis?

1) Acoustic neuroma 2) Chronic otitis media - This patient has a cholesteatoma which is a complication of chronic otitis media. Chronic otitis media is a persistent, chronically draining (>6 wk), suppurative perforation of the tympanic membrane. Symptoms include painless otorrhea with conductive hearing loss. Other complications include development of aural polyps along with other infections. Cholesteatoma is a destructive and expanding growth consisting of keratinizing squamous epithelium in the middle ear or mastoid process. They can cause problems because of their erosive and expansile properties resulting in destruction of the bones of the middle ear. A hallmark symptom of cholesteatoma is painless otorrhea. 3) Malignant otitis externa 4) Mastoiditis 5) Otosclerosis

A 48-year-old woman comes to the office for evaluation of ringing and progressive hearing loss in her right ear, and headache for the past two months. She has also noticed some difficulty maintaining her balance over the past month. Results of hearing tests show a high-frequency sensorineural hearing loss on the right. Physical examination shows slightly decreased sensation over the right side of the face. The remainder of the examination, including that of the ear, shows no other abnormalities. Which of the following is the most likely diagnosis?

1) Acoustic neuroma Acoustic neuroma (AKA: vestibular schwannoma) is a benign tumor of vestibular schwann cells of the 8th cranial nerve. Unilateral hearing loss is the initial symptom in approximately 90% of affected individuals. Hearing loss is usually gradual, although in some cases, it can fluctuate (worsen and then improve). Common additional findings include tinnitus, difficulty understanding speech disproportionate to degree of hearing loss, and dizziness or imbalance. Enlargement of the neuroma may also impact other cranial nerves, and cause additional symptoms such as facial numbness or tingling. 2) Bell palsy 3) Meniere's disease 4) Otosclerosis 5) Presbycusis

Which of the following can cause sudden onset, unilateral, painless vision loss? 1) Acute angle closure glaucoma 2) Cavernous sinus thrombosis 3) Central retinal artery occlusion 4) Orbital cellulitis 5) Ultraviolet keratitis

1) Acute angle closure glaucoma - Acute angle closure glaucoma is associated with severe pain, nausea, vomiting, and headache. Seeing rainbows or halos around the lights can also be an associated symptom. 2) Cavernous sinus thrombosis - Cavernous sinus thrombosis typically presents with a headache, unilateral or sometimes bilateral swelling, fever, and inability to move the involved eye. 3) Central retinal artery occlusion - Central retinal artery occlusion characteristically presents with unilateral sudden, and painless, non-progressive vision loss . Associated symptoms of headaches, jaw claudication, scalp tenderness, muscle and joint pain, appetite loss and fever can be identified. 4) Orbital cellulitis - Orbital cellulitis is a dangerous infection that can result in lasting sequela. The typical presentation is general overall feeling of being ill, inflammation to affected eye, decreased vision, and pain on movement of affected eye. 5) Ultraviolet keratitis - Ultraviolet keratitis is an effect of the eye being exposed to ultraviolet rays for significant duration of time. It is characterized as a foreign body sensation with photophobia and pain. There can also be an associated decreased visual acuity.

A 65-year-old man is brought to the emergency room because of sudden vision loss in his right eye earlier that day. He denies any trauma or any pain in the eye. On fundoscopy, the retina appears pale with a central area of redness. Which of the following best explains this patient's history and physical exam findings?

1) Acute angle closure glaucoma - Acute angle closure glaucoma results in acute vision loss, but typically the patient experiences pain. It is caused by the rapid narrowing of the anterior chamber angle, thereby preventing drainage of aqueous fluid through the trabecular meshwork. This is an ophthalmic emergency. Patients will have a red, painful eye and fixed mid-dilated pupil. Nausea and vomiting can also occur. Emergent evaluation by an ophthalmologist is critical. 2) Central retinal artery occlusion - This patient's signs and symptoms are most consistent with central retinal artery occlusion (CRAO). CRAO is usually due to an embolic event, and patients may even have multiple transient occurrences of vision loss. Since it is arterial occlusion, it is more worrisome than CRVO. The classic description of fundoscopic findings is a "pale retina with a cherry red spot". Ophthalmology should be promptly consulted, as recovery of vision is poor. 3) Central retinal vein occlusion - Central retinal vein occlusion (CRVO) will also present with acute painless vision loss, but the retina will have the appearance of the classic "blood and thunder", with diffuse retinal hemorrhage, dilated, tortuous veins, and optic disc edema. CRVO is generally caused by thrombosis, and CRAO by embolism. 4) Hypertensive retinopathy - Patients with hypertensive retinopathy may or may not notice any visual symptoms. Associated fundoscopic examination findings include retinal hemorrhages, exudates, and papilledema. 5) Open angle glaucoma - Though open angle glaucoma results in a painless vision loss, it is a more gradual onset than with CRAO. Glaucoma is associated with increased intraocular pressure, which then leads to optic nerve damage. Patients will first notice loss of peripheral vision. Risk factors include age, family history, hype

A 78-year-old female comes to the clinic for an annual check-up. She reports that over the past year, her vision is gradually worsening. She also admits that colors seem less vibrant and that glare bothers her, especially at night. Physical exam reveals opalescent changes to the lens. Which of the following is her most likely diagnosis?

1) Acute angle-closure glaucoma - Acute angle-closure glaucoma leads to sudden, painful vision loss secondary to increased intraocular pressure. 2) Cataracts - Cataracts cause a clouding of the lens which leads to blurred vision and potential blindness. Symptoms occur gradually. The only cure is surgical removal of the lens, followed by installation of an artificial lens. 3) Macular degeneration - Macular degeneration presents with gradual vision loss secondary to damage to photoreceptors 4) Pinguecula - A pinguecula is a yellowish, slightly raised thickening of the conjunctiva. They rarely interfere with vision unless they intrude on the cornea. 5) Retinopathy - Retinopathy encompasses many diseases which cause vision loss secondary to retinal damage.

A 26-year-old man has had a painful bloody spot in his left eye for the past day. He says his toddler hit him with a toy while they were playing yesterday, and it hurt quite a bit. He noticed he was having trouble seeing out of his left eye within an hour or so of the trauma. The patient wears contacts but chose to wear glasses today because he is sensitive to light. Which of the following is the most likely cause of the ocular pain in this patient?

1) Acute angle-closure glaucoma 2) Bleeding in the conjunctiva 3) Bleeding of the anterior chamber of the eye Hyphema is often associated secondary to trauma and presents as blood in the space between the iris and cornea. Intraocular pressure must be assessed, and the eye must be rested to decrease swelling and prevent additional bleeding. The blood may need to be evacuated to prevent corneal damage. 4) Keratitis 5) Metal in the cornea

A 42-year-old woman who works as a flight attendant comes to the urgent care clinic because she has had persistent congestion and mild pain in her right ear for the past 5 days. She is otherwise healthy and has no history of recent upper respiratory tract infection. Vital signs are within normal limits. Cardiac examination and examination of the head, eyes, ears, nose, and throat show no abnormalities. Which of the following is the most likely diagnosis?

1) Acute otitis externa 2) Acute otitis media 3) Eustachian tube dysfunction The eustachian tube equalization of pressure in the middle ear can become dysfunctional with repeated changes in ambient pressure, such as when flying on an airplane. 4) Foreign body 5) Sinusitis

A 13-year-old boy is brought to the office by his mother because he has had a runny nose with clear discharge and sneezing for the past month. Physical examination shows periorbital darkened circles and a pale boggy nasal mucosa. Which of the following is the most likely diagnosis?

1) Acute sinusitis - Acute sinusitis is defined as symptoms of less than four weeks in duration, and usually manifests in adults as more than 7 days of nasal congestion, purulent rhinorrhea, postnasal drip, and facial pain and pressure. 2) Allergic rhinitis - Symptoms of allergic rhinitis include nasal congestion, sneezing, rhinorrhea, itching of eyes, nose and palate, post-nasal drip, frequent throat clearing, cough, fatigue and malaise, both of which are especially in common in children. Signs include clear rhinorrhea, bluish or pale swelling of nasal mucosa, ocular findings (watery discharge, swollen conjunctivae, scleral injection, periorbital 'dark circles'), allergic 'shiners', and nasal crease. 3) Chronic sinusitis - Chronic sinusitis is defined as symptoms lasting longer than eight weeks, and can cause more indolent symptoms that persist for months. Nasal congestion, and postnasal drainage are the most common symptoms of chronic sinusitis. 4) Rhinitis medicamentosa - The characteristic presentation of this condition involves nasal congestion (without rhinorrhea), postnasal drip, or sneezing following several days of decongestant use. 5) Vasomotor rhinitis - Vasomotor rhinitis is characterized by symptoms of nasal obstruction, rhinorrhea, and congestion. These symptoms are exacerbated by certain odors (e.g. perfumes, cigarette smoke, paint fumes, inks), alcohol, spicy foods, emotions, and environmental factors (e.g. temperature, barometric pressure changes, bright lights).

A 42-year-old man who works in a factory comes to the onsite medical clinic immediately after he sustained an eye injury. He says he was hammering on a piece of metal when he felt something fly into his right eye. He was not wearing the recommended eye protection while working. On physical examination, visual acuity is 20/20. Fluorescein staining of the eye shows a small foreign body, which is successfully removed. Erythromycin ointment is applied. Which of the following is the most appropriate next step?

1) Administration of a tetanus booster - A tetanus booster should be administered to any patient with a penetrating foreign body in the eye who is not up-to-date on their tetanus vaccinations. 2) Application of an eye patch 3) Cromolyn drops 4) Installation of tropicamide drops 5) Use of pH paper

What condition should be suspected in a patient who received blunt trauma to the right eye and on an exam, has limited extraocular movement?

1) Blowout fracture with entrapment - Orbital trauma and limited extraocular movement is concerning for a blowout fracture with extraocular muscle entrapment. 2) Hyphema 3) Intracranial hemorrhage 4) Maxillary fracture with impinged nerve 5) Nasal septal fracture

A 4-year-old male is brought into the clinic by his mother because he has had ear pain and a fever for the last two days. You diagnose an acute otitis media. Mom is frustrated because this is the fourth time in the past year that he has received that diagnosis. In addition to antibiotic treatment, what is the next best step in the management of this patient?

1) Admit for surgical consultation 2) Discharge child with ENT referral for possible myringotomy tube placement - This is the next best step in the management of chronic otitis media. Otitis media is considered chronic if there are 3 documented episodes in 6 months, or 4 episodes in one year. These children should be sent to an ENT for evaluation and treatment. 3) Discharge with follow-up with pediatrician as scheduled next year 4) Order an MRI to evaluate for craniofacial anomalies 5) Reassure mom that it is very common for children in his age group to have multiple episodes, and no treatment is required

A 66-year-old male presents to your clinic complaining of painless hoarseness for the past month. He admits to a heavy smoking history, but this hoarseness seems worse than usual, enough so that he has not smoked for the past two weeks thinking that it would help, though it has provided no relief. He denies fever, cough, or sore throat. What is the next step in the care of this patient?

1) Advise the patient to maintain hydration and voice rest and his condition should resolve within the next week - A patient with a simple case of laryngitis would be advised to maintain hydration and voice rest. However, this patient has gone the recommended time frame without relief and now requires additional attention. 2) Counsel him to continue abstaining from tobacco use and his condition should resolve within the next few weeks - The use of tobacco can lead to voice changes by various mechanisms. One mechanism is Reinke's edema where the upper layer of the vocal cords can become edematous and cause a hoarseness which can improve once smoking has ceased. Another cause can be cancer of the vocal cords which would not resolve with abstaining from tobacco use. 3) Order a CT scan of the chest - A CT scan of the chest can be ordered once there is a diagnosis of vocal cord cancer to determine if there has been spread. The test of choice would be a laryngoscopy. 4) Prescribe a course of antibiotics to treat his infection - Antibiotics would have no role as the patient denies any signs of infection. 5) Refer to ENT - All patients that have persistent hoarseness (>2 weeks duration) should be referred to ENT to rule out neoplastic processes like laryngeal cancer.

A 66-year-old man comes to the office for evaluation of gradual central vision loss over the past two years. Amsler grid testing shows several blurry spots in the field of vision. Fundoscopy reveals discrete yellow deposits on the retina. Which of the following best explains this patient's symptoms and physical exam findings?

1) Age-related macular degeneration - Age-related macular degeneration (AMD) is the most common cause of irreversible central vision loss in elderly patients. Visual changes can often be detected with an Amsler grid, and dilated funduscopic findings are diagnostic. 2) Amaurosis fugax 3) Anterior uveitis 4) Glaucoma 5) Posterior uveitis

An 18-month-old child comes to the clinic with her mother for a well-child check. She undergoes screening for strabismus. This is performed in young children in order to prevent which of the following conditions?

1) Amblyopia - Amblyopia is a condition in which visual acuity in one or both eyes is permanently compromised because of abnormal visual experiences during the development of normal eyesight. This can be caused by strabismus (misalignment of the eyes). If detected, strabismus can be treated with refractive error correction, occlusion therapy, visual training exercises, and surgery. 2) Astigmatism - Astigmatism is a condition caused by an irregularly shaped cornea. Unlike those with a spherical cornea, light can not be focused to a single point. Treatment involves a cylindrical lens. 3) Cataracts - Pediatric cataracts can be inherited, or due to other diseases, such as intrauterine infection, metabolic disease, and chromosomal abnormalities. Infants with cataracts may have an asymmetric red reflex, leukocoria, or strabismus on physical examination. Management is focused on maintaining normal visual development. 4) Myopia - Myopia, or nearsightedness, occurs when light focuses anterior to the retina, resulting in a blurry imaging reaching the retina. The most common age for development is in adolescence. It is corrected with a concave lens. 5) Retinal detachment - A retinal detachment can lead to permanent vision loss. It occurs more often in adults, people with myopia, a history of eye trauma, and cataract surgery. Patients will present with floaters, flashes of light, and sudden vision loss, like a curtain coming down over the eye. Urgent ophthalmology consult is warranted.

A 28-year-old woman comes to the office because of right-sided facial pain and pressure, nasal congestion, runny nose, fever, and maxillary dental pain that has worsened over the past two days. She states that she has been sick with a cold for the past seven days, and is now worse. Physical examination shows tenderness to percussion over the right maxillary sinus and purulent nasal secretions. Transillumination of the right maxillary sinus suggests the presence of an air-fluid level. Which of the following is the most appropriate treatment for this patient's condition?

1) Amoxicillin-clavulanate - Amoxicillin-clavulanate is the most appropriate initial therapy for patients with acute bacterial rhinosinusitis who do not have risk factors for resistance. Clavulanate improves coverage for ampicillin-resistant H. influenzae as well as M. catarrhalis over that of amoxicillin alone. 2) Azithromycin 3) Cephalexin 4) Levofloxacin 5) Trimethoprim-sulfamethoxazole

A 65-year-old man comes to the office for routine annual physical examination. He has been healthy but says that he has noticed a nonpainful white patch on his tongue that seems to have enlarged over the past four months. Attempts at removing the patch with his toothbrush have not been successful. Physical examination shows a 1-cm adherent, bright white plaque on the right lateral margin of the tongue that is sharply defined and slightly raised above the surrounding mucosa. Results of a punch biopsy are negative for neoplasm. A history of which of the following is a known risk factor for the lesion noted in this patient?

1) Amyloidosis 2) Cigarette smoking Years of cigarette smoking expose the tongue to irritant chemicals and heat and can cause the development of leukoplakia. 3) Excessive chewing of gum 4) Herpes simplex virus infection 5) Human papillomavirus infection

A 23-year-old man comes to the emergency department because of sore throat, and difficulty and pain with swallowing for the past day. He states that he has had a cold for the previous three days. Temperature is 38.4º C (101.2º F). On general appearance, you note that he speaks with a muffled, "hot-potato" voice, and his body posture is held in a tripod position. Physical examination shows mild drooling and tenderness to palpation over the larynx. Which of the following is the most likely diagnosis?

1) Angioedema 2) Croup 3) Epiglottitis - Typical clinical manifestations of epiglottitis include acute occurrence of high fever, severe sore throat, difficulty in swallowing, sitting up and leaning forward (tripod) position in order to enhance airflow, and muffled (hot potato) voice. There is usually drooling because of difficulty, and pain on swallowing. Acute epiglottitis usually leads to generalized toxemia. 4) Infected branchial cleft cyst 5) Laryngitis

A 73-year-old man is brought to the emergency department by ambulance because he has a profuse nosebleed that he has been unable to stop at home despite pinching both nostrils closed for two 15-minute intervals. The emergency medical technicians packed his anterior nares bilaterally at the scene. His medical history includes atrial fibrillation, for which he takes rivaroxaban and metoprolol. Temperature is 37.0ºC (98.6ºF), pulse rate is 62/min, respirations are 18/min, and blood pressure is 156/84 mmHg. Physical examination shows blood in the posterior pharynx. Which of the following arteries is the most likely source of this patient's bleeding?

1) Anterior ethmoid 2) Lateral nasal 3) Sphenopalatine - This patient has posterior epistaxis which can cause profuse, hard to control nosebleeds, usually requiring inpatient admission, ENT consultation, and posterior nasal packing. Risk factors include anticoagulant therapy, hemophilia, and hypertension. Posterolateral branches of the sphenopalatine artery are common sources of bleeding in posterior epistaxis. 4) Sublingual 5) Superior labial

A 16-year-old girl is brought to the emergency department because she has a one-week history of facial pain and rhinorrhea and a 24-hour history of eye pain, tenderness, and double vision. Temperature is 38.0°C (100.4°F), pulse rate is 88/min, and blood pressure is 117/68 mmHg. Physical examination shows pain with movement of the right eye. Findings on eye examination are shown. What is the most appropriate next step in the management of this patient?

1) Antibiotic therapy 2) CT scan of the orbit and sinuses with contrast - This patient has orbital cellulitis. She has a history of sinusitis based on one week of facial pain and rhinorrhea which is the most common preceding illness. Red flag symptoms of eye pain, double vision, fever, and pain with ocular movements require further workup with a CT scan to rule out complications such as abscess. She should also have an ophthalmologic evaluation in the future and serial eye examinations. 3) Facial x-ray study 4) Referral for outpatient ophthalmologic evaluation 5) Referral to an otolaryngologist for sinus endoscopy

A 42-year-old woman with a history of asthma is seen by her primary care provider for evaluation of a burning sensation of her tongue, gums, and palate that began two days ago. She denies fever, nasal congestion, headache, sore throat, cough, or shortness of breath. Current medications include a fluticasone inhaler daily and an albuterol inhaler as needed. She completed a 10-day course of amoxicillin/clavulanate four days ago for a dog bite on the left hand. Vital signs are normal. Physical examination shows erythema and white patches on the buccal mucosa, tongue, and palate. Her gums appear red and swollen. Scraping a white patch off of the buccal mucosa leaves a red, eroded area underneath. The wounds on her hand are healing and there is no redness, swelling, or tenderness. The remainder of the examination shows no abnormalities. Which of the following best explains this patient's history and physical findings?

1) Aphthous stomatitis - Aphthous stomatitis (canker sores) is a common disorder of the oral mucosa characterized by one or more shallow, painful ulcers that have a yellow pseudomembrane, and an erythematous border. The ulcers are confined to nonkeratinized mucosa of the mouth. Thus, these lesions are not found on the dorsum of the tongue or palate. 2) Herpes labialis - Herpes labialis (also called "cold sores" or "fever blisters") is an infection most commonly caused by Herpes simplex virus type 1. Painful vesicles associated with erythema are typically distributed on the lips, gingiva, palate, and tongue. White patches are not typical. 3) Lichen sclerosus - Lichen sclerosus (LS) is a chronic inflammatory dermatosis that results in white plaques with epidermal atrophy and scarring. It primarily affects the genital areas. Lichen sclerosis usually begins as white papules that coalesce to form plaques. These plaques can then lead to fibrosis. 4) Oral candidiasis - Oral candidiasis, or thrush, is caused by an overgrowth of yeast (usually Candida albicans) in susceptible individuals. Patients at risk for thrush include neonates, those who have recently taken antibiotics or steroids, and those who are immunocompromised. The overgrowth of yeast leads to the desquamation of epithelial cells, and the accumulation of bacteria, keratin and necrotic tissue that forms a pseudomembrane, which may closely adhere to the mucosa. Scraping the membrane off may cause bleeding of the underlying mucosa. 5) Oral leukoplakia - Oral leukoplakia is a term used to characterize an adherent well-demarcated white patch or plaque that cannot be rubbed off. It is a diagnosis of exclusion, and usually there is no known etiology. However, tobacco use, alcohol consumption, and vitamin deficiencies have been implicated. Patients are asymptomati

A 54-year-old diabetic man comes to your clinic after being treated for a urinary tract infection with an antibiotic a week ago. He is now complaining of a sore throat and painful lesions in his mouth. On physical examination, you note white plaques that you are able to scrape off to reveal underlying erythema and friable oral mucosa. Which of the following best explains this patient's symptoms and physical exam findings?

1) Aphthous ulcers 2) Oral candidiasis - This presentation and description is consistent with oral candidiasis, or thrush. Thrush is characterized by white plaques on the tongue, buccal mucosa, or palate, that are easily scraped away, with underlying erythema. Remember, 'thrush will brush'. It occurs in immunocompromised patients, patients using corticosteroid inhalers, patients who use dentures, and in those recently treated with antibiotics. Treatment involves topical antifungal agents. 3) Oral leukoplakia 4) Oropharyngeal carcinoma 5) Streptococcal pharyngitis

A 3-year-old boy is brought to the clinic by his father because his nose has been making a wheezing sound on the left side with inspiration for the past hour. The father says that the patient was counting beads with his babysitter when he suddenly looked uncomfortable and began scratching his nose. Which of the following is the most appropriate initial step in management?

1) Application of an antibacterial cream 2) Direct visualization to remove the foreign body This is the treatment of choice for resolving a foreign body obstruction of the nose. Direct visualization ensures that the entire object will be removed with minimal trauma. 3) Observation 4) Surgical excision of the foreign body 5) Use of nasal saline spray

A 34-year-old woman comes to the urgent care clinic because of pain, redness, foreign body sensation, and photophobia of the left eye that have worsened over the past two days. She usually wears contact lenses but has been unable to do so for the past day. Physical examination of the left eye shows ciliary injection and pupillary constriction. Slit lamp examination following application of fluorescein stain shows a 3 mm defect in the cornea. Infiltrates with stromal inflammation in the central visual axis are present. Which of the following is the most appropriate next step in management?

1) Application of topical anesthetic and eye patch 2) Application of topical corticosteroid and eye patch 3) Emergent referral to an ophthalmologist - This patient has a corneal ulcer. All patients with corneal ulcers should be seen by an ophthalmologist within 12 to 24 hours. However, given antibiotic resistant patterns and that this particular ulcer which is large >2mm, central and involving the stoma, it requires an emergent ophthalmologic consultation for culture to guide the antibiotic regimen. 4) Intramuscular injection of ceftriaxone 5) MRI of the orbit

A 24-year-old transgender man comes to the urgent care 15 minutes after he got a chemical splashed in his eye at work. He works in a factory that manufactures cleaning supplies including products containing bleach. He says his eye hurts a lot and has been tearing. Temperature is 37.1°C (98.8°F), pulse rate is 80/min, respirations are 16/min, and blood pressure is 128/87 mmHg. Oxygen saturation is 98% on room air. Which of the following is the most appropriate initial step in the management of this patient?

1) Application of topical antibiotic - Following copious irrigation, topical antibiotics should be used to facilitate healing and prevent secondary infection. 2) Check visual acuity - While checking visual acuity is necessary to assess the amount of damage, it would not be done until after the eye has been appropriately irrigated. 3) Immediate referral to ophthalmology - Referral to an ophthalmologist would not occur until after irrigation and assessment have taken place. 4) Irrigation of the eye with water or normal saline - Irrigation is the initial step in any chemical injury to the eye. The pH of the eye should be returned and maintained at 7.0-7.2. 5) Review Material Safety Data Sheet - It is important to review the material safety data sheet (MSDS) for the chemical this patient was exposed to, but copious irrigation should be performed first.

A 3-year-old boy is brought to the clinic by his mother because he has been tugging on his right ear since last night and seems fussy. The patient is irritable but seems satisfactorily cooperative. On physical examination, no abnormalities are noted in the right pinna. The right external auditory canal is mildly erythematous, and a small button battery is visualized in the canal. The right tympanic membrane is slightly obscured but appears to be intact. Which of the following is the most appropriate initial step in management?

1) Apply cyanoacrylate to a cotton swab to remove the object 2) Irrigation of the ear canal 3) Removal of the battery with alligator forceps 4) Suctioning of the ear canal 5) Urgent referral to otolaryngologist

A 33-year-old man comes to the clinic because he has had a mass on his upper eyelid for the past month. He has no pain, and there has been no change in size. The patient has no history of fevers, blurred vision, or vision loss. Physical examination shows a 3-mm nodule on the upper eyelid that is not erythematous or tender to palpation. No induration of the nodule is noted. Which of the following is the most appropriate initial step in management? 1) Apply warm compresses 2) Cover the affected eye with a patch 3) Incise and drain the nodule 4) Observe and follow-up in two days 5) Refer to ophthalmology

1) Apply warm compresses - A chalazion is due to the inflammation and blockage of the meibomian sebaceous gland. Clinical presentation includes a painless mass located on the upper eyelid. Conservative management is the first-line treatment, which includes the application of warm compresses, cleansing the area regularly with baby shampoo, and eyelid massage. 2) Cover the affected eye with a patch - Application of an eye patch is an appropriate intervention for corneal abrasions > 5 mm for no more than 24 hours. Symptoms include a red, painful eye with a tearing sensation. Antibiotic drops and an ophthalmology referral within 24 hours are also a part of the treatment course. 3) Incise and drain the nodule - A hordeolum is an abscess located near the eyelid area most commonly caused by Staphylococcus aureus. Symptoms include a warm, erythematous, painful nodule on the eyelid. Conservative management is indicated with the application of warm compresses indicated as the first-line treatment. If no drainage occurs within 24 hours, then incision and drainage is indicated. 4) Observe and follow-up in two days - Observation is not recommended in the medical management of a chalazion. 5) Refer to ophthalmology - A referral to ophthalmology is indicated the management of a chalazion if symptoms persist for more than thirty days or if there is relocation of the nodule during the time period of using conservative treatment options.

A 33-year-old man comes to the primary care office because he has experienced a significant nosebleed on three or four occasions within the past month. He is able to control the bleeding within a few minutes with application of nasal pressure and denies excessive blood loss. The patient says that the nose bleeds started about five weeks ago when his spouse brought home a new kitten. His spouse insists that the kitten needs to sleep in their bedroom, but he admits to new onset of nasal congestion, clear rhinorrhea, sneezing, and watery eyes since the arrival of the kitten. The patient says that two weeks ago he went to the local drugstore and bought various over-the-counter allergy medications to help with his symptoms. Which of the following is the most important question to ask this patient?

1) Are you aware of any other environmental allergies? 2) Do any family members have similar symptoms? 3) Do you have any family history of a cat allergy? 4) Do you use a cool-mist humidifier at home? 5) What over-the-counter medication(s) are you using? A common side-effect of topical nasal glucocorticoids can be epistaxis due to both the drying effect of the medication and possible mechanical trauma from repeat spraying. This patient reports a new introduction of an environmental allergen and self-treatment with over-the-counter allergy medications.

Which of the following fundoscopic findings would be seen in diabetic retinopathy, but not hypertensive retinopathy? 1) Arteriolar narrowing and sclerosis 2) Arteriovenous nicking 3) Cotton wool spots 4) Neovascularization 5) Optic disc edema

1) Arteriolar narrowing and sclerosis - Arteriolar narrowing and sclerosis are associated with hypertensive retinopathy. These are described as "copper wiring" and 'silver wiring" respectively. These changes typically occur due to chronic hypertension. 2) Arteriovenous nicking - A/V nicking describes when an arteriole crosses a venule, resulting in a dilated, swollen vein. This finding is associated with hypertensive retinopathy. 3) Cotton wool spots - Cotton wool spots are fluffy, white lesions seen on the retina, indicating vascular insufficiency. These may be seen in both hypertensive and diabetic retinopathies. 4) Neovascularization - Neovascularization is characteristic of proliferative diabetic retinopathy. This occurs as a compensatory mechanism to revascularize areas with ischemic injury. However, these new vessels are fragile, and may cause distortion of the retina. Diabetics should aim for a HbA1C level of <6.5% to avoid any ophthalmic sequelae. 5) Optic disc edema - Optic disc edema is associated with hypertensive retinopathy, along with retinal hemorrhages and exudates. Macular edema is associated with diabetic retinopathy.

A 13-year-old girl is brought to the office because she has had worsening pain in the right ear with drainage from the ear over the past two days. The patient is an active member of her school's swim team. Physical examination of the ear shows diffuse inflammation and edema of the external auditory canal with significant debris and otorrhea. Which of the following is the most likely causal organism?

1) Aspergillus niger 2) Moraxella catarrhalis 3) Pseudomonas aeruginosa - The most common bacterial pathogens that cause otitis externa include P. aeruginosa and Staphylococcus aureus. 4) Staphylococcus epidermidis 5) Streptococcus pneumoniae

An 82-year-old man comes to the office for evaluation of tinnitus that has been present for the past week. He describes a buzzing and sometimes roaring sound in both ears that initially was intermittent, but is now more constant. He denies dizziness, headache, hearing loss, ear pain, or discharge. He has no history of trauma or exposure to loud noises. Medical history includes osteoarthritis, type 2 diabetes mellitus complicated by peripheral neuropathy, hypertension, and hyperlipidemia. Medications include 650 mg of aspirin taken every 4 hours, 500 mg of metformin twice daily, 20 mg of lisinopril once daily, 400 mg of gabapentin three times daily, and 20 mg of simvastatin once daily. Vital signs are normal. Examination of the ears, nose, and throat is normal. Cranial nerves are intact with symmetric function. There is no lateralization on Weber test, and air conduction is greater than bone conduction on the Rinne te

1) Aspirin - Salicylates, including aspirin, are analgesics used for the treatment of mild to moderate pain. Aspirin is an anti-inflammatory drug that is effective for the treatment of inflammation. Aspirin, and other non-steroidal anti-inflammatory drugs such as ibuprofen and naproxen can cause tinnitus. The development of tinnitus appears to be dose-related. In this scenario, discontinuation of the aspirin therapy would likely resolve this patient's tinnitus. Other drugs known to cause tinnitus are diuretics, drugs containing quinine, certain antibiotics (most notably aminoglycosides), and chemotherapeutic agents containing platinum. 2) Gabapentin - Tinnitus is not a known side effect of gabapentin. 3) Lisinopril - Tinnitus is not a known side effect of lisinopril. 4) Metformin - Tinnitus is not a known side effect of metformin. 5) Simvastatin - Tinnitus is not a known side effect of simvastatin.

A 27-year-old man is seen in the emergency department for right ear pain one day after being assaulted. He was struck once by an assailant's fist directly over his right ear. He does not have any other injuries or neck pain, and he did not have any loss of consciousness. Physical examination of the anterior auricle shows a tense, bulging cm mass underneath intact skin overlying the ear cartilage, along with swelling, deformity, and loss of auricular landmarks. The remainder of the examination shows no abnormalities. Even with appropriate intervention, which of the following is the most likely complication of this patient's condition?

1) Auricular abscess 2) Cauliflower ear - Auricular hematoma occurs when a blunt shearing force is sustained to the ear (e.g. boxing). These forces separate the underlying perichondrium, which is highly vascularized, from the cartilage itself. The result is formation of a hematoma between this space which disrupts blood flow to the cartilage. If left untreated, auricular hematoma may subsequently lead to cartilage necrosis with formation of a 'cauliflower' ear where healthy cartilage is replaced with fibrocartilage. 3) Cholesteatoma 4) Malignant otitis externa 5) Otosclerosis

A 58-year-old woman comes to the clinic because of dizziness. She reports the dizziness is episodic. She also complains of episodic hearing loss in her right ear as well as tinnitus and feeling of ear fullness. Temperature is 37.1°C (98.8°F), pulse rate is 80/min, respirations are 16/min, and blood pressure is 128/87 mmHg. Oxygen saturation is 98% on room air. Meniere disease is suspected. Which of the following classes of medication is most likely to be effective in the treatment of this condition?

1) Barbiturates 2) Beta-blockers 3) Diuretics Meniere disease is a chronic condition characterized by vertigo, sensorineural hearing loss, and tinnitus. The etiology is unknown. Goals of treatment are to reduce symptoms, improve hearing loss, and prevent disease progression. Diuretics are the mainstay of treatment, along with as-needed vestibular suppressants and antiemetics. Lifestyle modifications that can help include a low salt diet, reducing caffeine and alcohol, and smoking cessation. 4) Selective serotonin reuptake inhibitors 5) Tricyclic antidepressants

An 18-year-old man comes to the office because he had a red, painful bump on his upper outer eyelid yesterday that seems to have ruptured during the night. When he woke up this morning, he had a trace amount of purulent discharge draining from his eye, but the pain had mostly resolved. Which of the following is the most likely diagnosis?

1) Basal cell carcinoma 2) Episcleritis 3) Hordeolum 4) Pterygium 5) Squamous cell carcinoma

A 64-year-old man with a history of hypertension, hyperlipidemia, and smoking, comes to the office because of recurrent mild-to-moderate vertigo over the past month. He rarely has nausea. He denies recent trauma, ear pain, hearing loss, or vision changes. Physical examination of the eyes shows vertical nystagmus that is not suppressible. Neurologic examination shows ataxia, and an inability to perform a finger-to-nose test bilaterally. Which of the following is the most likely diagnosis?

1) Benign paroxysmal positional vertigo 2) Labyrinthitis 3) Meniere's disease 4) Perilymphatic fistula 5) Vertebrobasilar insufficiency Central vertigo results from dysfunction of the central connections of the vestibular apparatus including the vestibular nuclei in the brainstem and their connections, especially to the cerebellum. This case describes a central cause of vertigo. Central vertigo is characterized by: indolent vertigo, not necessarily positionally related, nystagmus that is not suppressible, and neurologic deficit (particularly ataxia). This patient demonstrates cerebellar deficits, and a central cause such as vertebrobasilar insufficiency or tumor, must be high on the differential diagnosis list.

A 32-year-old man comes to the office because he has had a red irritated eye for the past few weeks. During this time, he has had eye matting in the morning and dry eyes, and his eyelashes have been clumping together and several have fallen out. The patient tells you that he has been using dandruff shampoo more often recently due to an increase in the amount of dandruff he is experiencing. Physical examination shows a uniformly red, swollen lid margin with a clear conjunctiva. No discharge is noted. Which of the following is the most likely diagnosis?

1) Blepharitis - Blepharitis is characterized by red, swollen, or itchy eyelids, a gritty or burning sensation, crusting or matting of the eyelashes upon waking from sleep, excessive tearing, and light sensitivity. Blepharitis can be acute such as blepharitis caused by allergens or irritants, or chronic such as with seborrheic dermatitis or rosacea. Typical physical examination will demonstrate red, pink or irritated eyelids, which are often associated with crusting. The eyelids should be carefully examined for signs of chronic inflammation, such as misdirected eyelashes, loss of eyelashes, loss of pigmentation of eyelashes, and abnormal growth of eyelashes from meibomian gland openings. Chronic untreated blepharitis can result in entropion or ectropion of the lid, corneal erosions, corneal nodules, corneal ulcers, and corneal scarring. 2) Chalazion - This typically presents as a painless localized rubbery nodule or eyelid swelling without inflammation. 3) Kaposi sarcoma - This is a painless vascular lesion caused by a virus that appears red or purplish, seen most often in patients with AIDS and rarely, in men from the Mediterranean. 4) Seborrheic keratoses - This is described as a well-demarcated, round or oval lesion with a dull, verrucous surface and a "stuck-on" appearance. They are benign and slow-growing lesions consisting of a proliferation of immature keratinocytes. 5) Xanthelasma - This consists of cholesterol-filled, soft, yellow plaques that usually appear on the medial aspects of the upper and lower eyelids. They are often an indicator of elevated cholesterol levels.

A 4-year-old female is brought into the clinic by her mother, complaining of ear pain for the last two days. Mom reports that she has also had a fever, runny nose, and sinus congestion. Upon otoscopic evaluation, you note the tympanic membrane is erythematous and bulging, and it is hard to visualize landmarks. What is the most likely causative organism in this patient's condition?

1) Borrelia burgdorferi - This is the causative organism in Lyme disease. 2) Escherichia coli - E. coli is more common in neonates. 3) Haemophilus influenzae - H. influenzae is the second most common causative organism (but only roughly 20% of cases) in acute otitis media in children of all ages. 4) Moraxella catarrhalis - M. catarrhalis and H. influenzae are alternate causes of AOM, but not the most common. 5) Streptococcus pneumoniae - Streptococcus pneumoniae is the most common causative organism in acute otitis media in children of all ages.

A 5-year-old boy is brought to the office by his mother because she has noticed a persistent lump in his neck for the past month. Physical examination shows a midline 2 cm rubbery mass located just inferior to the hyoid bone. When he is asked to swallow or protrude his tongue, the mass transiently moves upward. Which of the following is the most likely explanation of these findings?

1) Brachial cleft cyst 2) Goiter 3) Normal contents of the carotid sheath 4) Submental lymph node 5) Thyroglossal duct cyst Thyroglossal duct cysts are the most common congenital neck cyst. They are typically located in the midline (~70% are located within 2 cm of the midline), and are the most common midline neck mass in young patients. Thyroglossal duct cysts typically present during childhood (90% before the age of 10), or remain asymptomatic until they become infected. They account for 70% of all congenital neck anomalies, and are the second most common benign neck mass, after lymphadenopathy. Presentation is typically either as a painless rounded midline anterior neck swelling or, if infected, as a red warm painful lump. It may move with swallowing, and classically elevates on tongue protrusion. Thyroglossal duct cysts are epithelial lined cysts that form from failure of normal developmental obliteration of the thyroglossal duct during development (8th-10th gestational week), and can thus occur anywhere along the course of the duct.

A child presents to your clinic with an acute otitis media infection. Which of the following is often found to precede this patient's current condition? 1) Bronchitis 2) Dental abscess 3) Pneumonia 4) Sinusitis 5) Upper respiratory infection

1) Bronchitis - Respiratory infections, such as bronchitis, don't usually precede AOM. 2) Dental abscess - Dental abscesses are more likely to precede sinusitis. 3) Pneumonia - Respiratory infections, such as pneumonia, don't usually precede AOM. 4) Sinusitis - Sinusitis can precede AOM, but it is much more common for a URI to precede AOM. 5) Upper respiratory infection - URI's are often found to precede AOM.

Significant or irritating entropion is sometimes surgically corrected to prevent which of the following?

1) Buildup of oils in the meibomian glands 2) Corneal damage from chronic trichiasis - Chronic rubbing of the eyelashes over the cornea due to inward turning of the eyelid can cause significant damage or ulceration and lead to infections that can compromise vision. 3) Formation of cataracts 4) Formation of chalazions 5) Increased intraocular pressure

A 15-year-old girl is brought to the office because she has had worsening pain in the right ear with drainage from the ear over the past two days. The patient is an active member of her school's swim team. Which of the following would be an unexpected physical examination finding in this patient? 1) Bulging red tympanic membrane 2) Canal clogged by debris 3) Conductive hearing loss 4) Ear pain elicited with manipulation of the tragus 5) Edematous external ear canal

1) Bulging red tympanic membrane - This patient has otitis externa which is very common in swimmers. The finding of a bulging and erythematous TM is more indicative of otitis media. 2) Canal clogged by debris 3) Conductive hearing loss 4) Ear pain elicited with manipulation of the tragus 5) Edematous external ear canal

A 15-year-old girl is brought to the office because she has had worsening pain in the right ear with drainage from the ear over the past two days. The patient is an active member of her school's swim team. Which of the following would be an unexpected physical examination finding in this patient?

1) Bulging red tympanic membrane This patient has otitis externa which is very common in swimmers. The finding of a bulging and erythematous TM is more indicative of otitis media. 2) Canal clogged with debris 3) Conductive hearing loss 4) Ear pain elicited with manipulation of the tragus 5) Edematous external ear canal

A 14-year-old male presents to the emergency department after being struck in the left eye with a lacrosse stick during practice. He is complaining of pain in and around the eye and some double vision. Physical exam reveals restricted upwards gaze on examination of his extraocular movement. Which of the following studies is most appropriate for this patient?

1) CT of the orbits - This patient has a presumed blowout fracture. CT scan of the orbits is the best test to diagnose an orbital blowout fracture and can also show important associated injuries such as extraocular muscle entrapment, orbital hematoma or globe rupture. 2) Cervical spine x-rays - Cervical spine x-rays are beneficial when there is significant trauma to the face to rule out cervical spine injury. This patient has a suspected orbital blowout fracture and needs a CT of the orbits. 3) Schiotz tonometry - This test is used to measure intraocular pressure. This patient has a suspected orbital blowout fracture and needs a CT. 4) Slit lamp examination - A slit lamp provides an illuminated, magnified w of the anterior portion of the eye. This exam can be used to detect abnormalities such as retinal detachment, macular degeneration, corneal abrasions and foreign bodies. 5) X-ray of the orbits - While an x-ray of the orbits would detect a fracture, a CT scan remains the test of choice due to its sensitivity and ability to detect associated injuries.

A 15-year-old boy is brought to the emergency department by his parents after he was struck bluntly in the right eye by an opponent's hand during a basketball game one hour ago. The patient has pain and loss of vision in the right eye. He does not wear contacts. Physical examination of the right eye shows conjunctival hemorrhage, enophthalmos, and a teardrop-shaped pupil. Visual acuity on the right is limited to light perception only. Which of the following is the most appropriate initial intervention?

1) Cycloplegic eye drops 2) Eye patch 3) Observation 4) Rigid eye shield - This patient's mechanism of injury and physical examination are consistent with globe rupture. Globe rupture is an ophthalmologic emergency. A rigid eye shield should be placed immediately to protect the eye from pressure while awaiting surgical consultation. 5) Tonometry

A 34-year-old patient who identifies as gender nonconforming comes to the office because of increasing nasal congestion on the right side of their nose over the past four months. They state that the congestion is positional, and is usually worse when they lay on the left side. Physical examination shows a small, fleshy translucent mass on the inferior turbinate of the right nasal cavity. The remainder of the examination shows no abnormalities. Which of the following best explains this patient's symptoms and physical exam findings?

1) Cancer of the nasal cavity 2) Deviated nasal septum 3) Hypertrophic turbinates 4) Nasal polyps Nasal polyps are fleshy outgrowths of the nasal mucosa that form at the site of dependent edema in the lamina propria of the mucous membrane. Conditions that predispose patients to the formation of these polyps include allergic rhinitis, acute and chronic infections, and cystic fibrosis. Nasal polyps are strongly associated with aspirin allergy, sinus infections, and asthma. Symptoms include nasal obstruction, postnasal drainage, congestion, sneezing, rhinorrhea, anosmia, hyposmia, facial pain, and ocular itching. The diagnosis is generally made on physical examination which shows teardrop-shaped, mobile, grey, smooth and semi-translucent polypoid masses that when mature, resembles a peeled seedless grape. 5) Nasal vestibulitis

Which of the following is consistent with an acoustic neuroma?

1) Causes vertigo, ataxia, tinnitus, hearing loss - Acoustic neuromas affect the nerves that run from the inner ear to the brain thus causing vertigo, ataxia, tinnitus, and hearing loss. 2) Cerebral tentorial herniation and brainstem compression 3) Malignant tumor of the acoustic nerve 4) Rapidly progressive 5) Treated with radiation

Which of the following is shown in this fundoscopic view?

1) Central retinal artery occlusion - Central retinal artery occlusion (CRAO) occurs when blockage of the retinal artery occurs. Classically, it is seen on fundoscopic examination as a cherry red spot. 2) Cotton wool spots - Cotton wool spots appear as white, fluffy spots on the retina and are the result of damage to nerve fibers typically caused by diabetes and hypertension. 3) Normal fundus - This is a normal fundoscopic image. 4) Papilledema - Papilledema is swelling of the optic disc secondary to increased intracranial pressure. 5) Retinal detachment - A retinal detachment occurs with the retina pulls away from the underlying epithelium.

Which of the following is shown in this fundoscopic view? 1) Central retinal artery occlusion 2) Cotton wool spots 3) Normal fundus 4) Papilledema 5) Retinal detachment

1) Central retinal artery occlusion 2) Cotton wool spots 3) Normal fundus 4) Papilledema 5) Retinal detachment

A 37-year-old woman comes to the office because she has had an intermittent sensation of ear fullness and feeling off-balance during the past week. The symptoms occur when she turns her head, sits up, or stands. Although each episode has lasted only a couple of minutes, they make the patient feel unsteady and are disruptive to her day. The patient says that her hearing has felt muffled, but she has not had fever, headache, visual changes, cough, congestion, or postnasal drip. She smokes two packs of cigarettes per day. Temperature is 37.3°C (99.2°F), pulse rate is 74/min, respirations are 16/min, and blood pressure is 142/74 mmHg. Physical examination shows moderate bogginess of the nasal conchae. The tympanic membranes are a dull bluish-gray color. In addition to smoking, which of the following is an additional risk factor for developing this syndrome?

1) Chronic rhinosinusitis This patient has eustachian tube dysfunction as noted by sensations of fullness, muffled hearing, vertigo, and bluish discoloration of the tympanic membranes consistent with effusion. Chronic sinusitis and smoking have been linked to increased risk of eustachian tube dysfunction. 2) Frequent swimming 3) History of nasal fracture 4) History of tympanic membrane rupture 5) Self-limiting pine allergy

A 15-year-old patient who identifies as non-binary comes to the clinic for evaluation of left eye pain. They say they slept in contact lenses last night and woke up with severe pain in the left eye. They have a gritty feeling in the eye and has difficulty keeping the eye open due to photophobia. On physical examination, the left eye is tearing profusely and the conjunctiva is injected. The pupil is round but miotic. Visual acuity is 20/20. Fluorescein examination is positive for a round area of uptake. No infiltrate or ulcer is noted on the cornea. Which of the following is the most appropriate treatment?

1) Ciprofloxacin drops - The most likely diagnosis is a corneal abrasion. Because this patient wears contacts, the chosen antibiotic must cover Pseudomonas species. Ciprofloxacin, ofloxacin, or gentamicin are all reasonable options. 2) Corticosteroid drops - Corticosteroid drops are not indicated for a corneal abrasion. They can impede healing and increase the risk of secondary infection. Ophthalmic corticosteroids must be used cautiously, as overuse can lead to cataracts. 3) Erythromycin ointment - A corneal abrasion in a contact lens wearer must be treated for any potential Pseudomonas infection. Erythromycin does not provide enough coverage for Pseudomonas, and it is an unacceptable antibiotic choice. Contact lens wearers are at an increased risk for pseudomonas keratitis, which can cause loss of vision. 4) Sulfacetamide drops - A corneal abrasion in a contact lens wearer must be treated for any potential Pseudomonas infection. Sulfacetamide does not provide enough coverage for Pseudomonas, and it is an unacceptable antibiotic choice. Contact lens wearers are at an increased risk for Pseudomonas keratitis, which can cause loss of vision. 5) Use of a pressure patch - A pressure patch should not be used to treat a corneal abrasion in an individual who wears contact lenses due to the risk of bacterial superinfection. A patch can warm the area under the patch and increase the risk of infection as well as prevent the patient from noticing changes in the eye. These consequences can cause the patient to lose vision in that eye. Patches may at times be recommended by ophthalmologists for large abrasions (greater than 10 mm) or for patients who have trouble not rubbing their eyes such as little kids and those with cognitive disabilities).

An 18-year-old transgender man comes to the office because he has had a sore throat and difficulty swallowing for the past three days. He has not had fever, cough, or rhinorrhea. He has several friends with similar symptoms. Temperature is 37.2°C (98.9°F), and pulse rate is 68/min. Oxygen saturation is 98%. Physical examination shows edema of the anterior lymph nodes and tonsils. Erythema of the posterior pharynx is noted. Which of the following is the most appropriate next step in diagnosis?

1) Complete blood count with differential - This laboratory test may be obtained in certain circumstances to better determine the cause of tonsillitis, but it is not usually indicated in diagnosis of strep throat. 2) Influenza swab - This patient's symptoms do not appear consistent with the diagnosis of influenza, a virus which usually involves the upper and/or lower respiratory tracts, including cough and rhinorrhea. 3) Monospot test - Anterior lymphadenopathy is most common for viral or bacterial pharyngitis, while posterior lymphadenopathy is usually indicated for infectious mononucleosis. Caused by the Epstein-Barr virus and also characterized by fever, fatigue, and possible splenomegaly, the severity of symptoms can vary greatly per patient and is initially diagnosed with a monospot test that provides results in one hour. 4) Rapid strep test - Acute pharyngitis can be viral or bacterial in nature. While Centor criteria estimates the likelihood that the pharyngitis is due to group A streptococci, a rapid strep test is the most appropriate first step in providing a diagnosis. This rapid antigen detection test is commonly used to help provide a quick diagnosis of bacterial pharyngitis caused by GAS for easy turn-around time. 5) Throat culture - Although a throat culture is definitive to ensure proper treatment and is more sensitive than a rapid strep test, this would not be the most appropriate first step, as it may take 24 to 48 hours to provide a result. A throat culture is usually obtained alongside a rapid strep test.

A 35-year-old woman comes to the office for evaluation of intermittent hearing loss over the past 12 months. Physical examination shows lateralization to the right ear with a Weber test, and bone conduction greater than air conduction in the right ear with a Rinné test. Which of the following is the most likely explanation for these findings?

1) Conductive hearing loss in both ears - This is characterized by absence of lateralization to either ear (i.e. midline) with Weber test, and bone conduction > air conduction of both ears with Rinné test. 2) Conductive hearing loss of the left ear - This is characterized by lateralization to the left ear with Weber test, and bone conduction > air conduction of the left ear with Rinné test. 3) Conductive hearing loss of the right ear - This is characterized by lateralization to the right ear with Weber test, and bone conduction > air conduction of the right ear with Rinné test. 4) Sensorineural hearing loss of the left ear - This is characterized by lateralization to the right ear with Weber test, and air conduction > bone conduction in both ears with Rinné test. 5) Sensorineural hearing loss of the right ear - This is characterized by lateralization to the left ear with Weber test, and air conduction > bone conduction in both ears with Rinné test.

A 2-year-old boy is brought to the office by his mother because she has noticed that the vision in his left eye has progressively deteriorated over the past month. Physical examination of the eye shows a white pupillary reflex in response to light. Funduscopic examination shows a white mass projecting from the retina. Which of the following is the most likely diagnosis?

1) Congenital cataract - Congenital cataracts (lens opacification) are usually diagnosed at birth. Unilateral cataracts are usually isolated sporadic incidents, but can be associated with ocular abnormalities, trauma, or intrauterine infection (e.g. rubella). Bilateral cataracts are often inherited, and associated with other diseases that require metabolic, infectious, systemic, and genetic workup. 2) Macular degeneration - Macular degeneration is a deterioration of the central portion of the retina, and is a leading cause of blindness in the US. Nonexudative (dry) age-related macular degeneration (ARMD) makes up 90% of patients diagnosed with ARMD. It is associated with the presence of drusen and no visual loss early in the disease, but often progresses to retinal atrophy and central retinal degeneration with loss of central vision. Nonexudative ARMD has a slower (decades) disease progression compared with exudative (wet) ARMD (months). 3) Ocular melanoma - Ocular melanoma is the most common primary cancer of the eye in adults. It occurs most often in lightly pigmented individuals, and the median age of occurrence is 55 years. 4) Retinitis pigmentosa - Retinitis pigmentosa is an inherited, degenerative disease of the eye that causes severe vision impairment from progressive degeneration of the rod photoreceptor cells in the retina. Initial symptoms appear independent of age, and can occur anytime from infancy to late adulthood. The initial symptoms are decreased night vision, and loss of the mid-peripheral visual field. 5) Retinoblastoma - Retinoblastoma is the most common primary malignant intraocular tumor in children. It originates from the retina, and can be unilateral or bilateral. It most commonly occurs in children less than 5 years of age. Whitening of the red reflex is the most common presentation fo

A patient comes to the clinic because he has had pain in the right eye for the past two days. He says that he wears contact lenses and typically sleeps with the lenses in place. Fluorescein staining shows uptake in a circular pattern at the 3 o'clock position. The conjunctiva and anterior chamber are clear. Which of the following is the most likely diagnosis?

1) Conjunctivitis - An infection of the conjunctiva leading to swelling and redness of conjunctiva. This patient presentation is not consistent with conjunctivitis 2) Corneal ulcer - A corneal ulcer is an open sore on the cornea that is painful and can lead to scarring. Corneal ulcers typically develop from bacterial or viral infections. 3) Globe rupture - A globe rupture can be detected by streaming fluorescein stain. Globe ruptures are a medical emergency and immediate ophthalmologic consultation is needed. 4) Hyphema - Hyphema consists of blood in the anterior chamber, typically caused by trauma. This patient's presentation is not consistent with hyphema. 5) Subconjunctival hemorrhage - Ruptured capillaries in the conjunctiva lead to a disconcerting blood appearance in the conjunctiva. There is no fluorescein uptake in a subconjunctival hemorrhage

A 29-year-old woman comes to the office for evaluation of her third sinus infection over the past six months. Her current symptoms of right-sided nasal discharge and fullness in the right side of her face are the same as her previous episodes and began three days ago. At her initial visit, she was treated with an antibiotic. At her second visit three months later, she was treated with intranasal steroids, nasal irrigation with saline, and decongestants. On both prior occasions, she had some, but not complete relief of her symptoms. Which of the following imaging studies is the most appropriate next step in the evaluation of this patient?

1) Coronal computed tomography - In this scenario the patient would benefit from imaging of her sinuses to look for mucosal disease, or anatomic and structural abnormalities. While magnetic resonance imaging (MRI) and computed tomography (CT) will both show mucosal disease, CT is the preferred modality because is offers better resolution of mucosal disease, and sinus ostial occlusion compared with MRI. CT is also less expensive. The coronal view provides excellent visualization of the ethmoid sinuses which are a common focus of chronic infection and inflammation. 2) Fluoroscopy 3) Positron emission tomography 4) Ultrasonography 5) Waters' view plain radiography

A 4-year-old girl is brought to the emergency department because she has had fever to 39.4°C (103.0°F) and worsening stridor and respiratory distress over the past five hours. The patient appears ill. She is sitting on her father's lap and leaning forward and drooling with her mouth open. Her mother says that she has not received any childhood vaccinations. X-ray study is shown. Which of the following is the most likely diagnosis?

1) Croup - Croup may be visualized on an AP neck x-ray (or chest x-ray) by what is referred to as the "steeple sign". The steeple sign is where subglottic tracheal narrowing produces the shape of a church steeple within the trachea itself. This patient has epiglottitis with a "thumbprint" sign seen on x-ray. 2) Epiglottitis - This patient has epiglottitis which can be a life-threatening emergency due to swelling of the epiglottitis leading to airway obstruction. A soft-tissue lateral neck x-ray may demonstrate the thumbprint sign as seen on this x-ray. 3) Peritonsillar abscess - A peritonsillar abscess can cause drooling but typically not stridor and the level of distress as this patient is in. It also has no typical radiographic findings associated with it. 4) Pertussis - Pertussis causes whooping cough and would be a concern in a patient who has not received immunizations. However, this is not the clinical picture of pertussis. It has no radiographic findings associated with it. 5) Uvulitis - Uvulitis is not visualized on x-ray.

A 21-year-old female, who had facial reconstructive surgery after a motor vehicle accident during the past year, presents to your clinic today complaining of a lump under her left eye. She reports that it has been growing steadily over the last few days, and has become more and more tender. Upon arrival, she has a temperature of 101.5º F (38.6º C), and on physical exam, you note swelling over the nasolacrimal sac that is erythematous, warm, and mildly fluctuant. What is the most likely diagnosis?

1) Dacryocystitis This presentation is consistent with a dacryocystitis, inflammation and possible infection of the nasolacrimal structures usually secondary to obstruction. This patient's history of facial surgery should increase your suspicion for this condition. 2) Erysipelas 3) Nasolacrimal duct obstruction 4) Periorbital cellulitis 5) Superficial cellulitis

A 34-year-old woman comes to the clinic with headaches, jaw stiffness, and pain that is exacerbated by chewing for 1 month. She states that she has a history of nighttime teeth grinding. Which of the following treatment modalities is indicated in the initial treatment of her condition?

1) Dental extraction 2) Mandibular nerve block 3) Oral contraception 4) Sumatriptan 5) Teeth guard Lorazepam, teeth guards, physical therapy referral, muscle relaxants are all first-line conservative treatments that may be attempted in this patient, who is suffering from TMJ dysfunction. Surgery may be employed after all other conservative measures have failed.

A patient comes to the urgent care clinic because he feels like something is trapped under his eyelid. He was trimming branches off of a tree in his yard a few hours ago and felt something fall into his eye. He is pretty sure that he was able to rinse out the debris but still felt like something was in his eye all day. Currently, he says that it is hard to open his eye and the sunlight is bothering him. Which of the following tests is most likely to determine the diagnosis?

1) Dilated eye examination 2) Examination of the eye with a pen light 3) Fluorescein staining and evaluation of the cornea under a cobalt blue light - Fluorescein staining will help identify a corneal epithelial defect that was likely caused by the foreign body before it was rinsed out. 4) Tonometry testing 5) Visual acuity testing with a Snellen chart

A 14-year-old boy is brought to the emergency department by his parents because he has had fever, headaches, and worsening bilateral facial and neck swelling over the past two days. He also has ear pain and pain when opening his mouth. Temperature is 38.0°C (100.4°F). Physical examination of the head and neck shows tenderness and swelling at the angle of the jaw bilaterally. Tympanic membranes are clear. Tonsils are pink, symmetric, and without exudate. Testes are swollen bilaterally. The result of a rapid strep test is negative. The result of a heterophile antibody test is negative. Serum amylase is 230 U/L. Which of the following is the most likely diagnosis?

1) Diphtheria - This patient has mumps parotitis. Diphtheria is extremely rare in the United States. Patients may present with a characteristic "bull neck" due to lymph node swelling, and a grey pseudomembrane can be seen on the tonsils. Diagnosis is by Gram stain or throat culture. 2) Mononucleosis - This patient has mumps parotitis. Patients with mononucleosis present with tender cervical lymphadenopathy. A positive heterophile antibody test is diagnostic of mononucleosis. 3) Parotitis - This patient has parotitis due to a mumps infection, the most common viral cause of parotitis. Males may also experience orchitis due to mumps infection. Elevated serum amylase levels are a clue to mumps parotitis diagnosis. Swelling of the parotid gland may begin unilaterally and then become bilateral. Fever, headache, otalgia, and trismus are common signs and symptoms of parotitis. 4) Peritonsillar abscess - This patient has mumps parotitis. Peritonsillar abscess is a deep neck infection that is usually unilateral. Patients present with a muffled "hot potato" voice, pharyngitis, dysphagia, and trismus. Unilateral swelling of the tonsil and contralateral deviation of the uvula are seen on physical examination. 5) Sjögren syndrome - This patient has mumps parotitis. Sjögren syndrome is an autoimmune disorder of exocrine glands and is a known cause of parotitis.

A 56-year-old man with a history of chewing tobacco use comes to your family medicine clinic because he noticed a white spot on the inside of his lip about a week ago. He denies pain at the site, oral discomfort, or hot or cold sensitivity. On physical examination, the lesion is on the inside of the right lip and extends slightly onto the buccal mucosa. The lesion remains intact upon inspection and scraping with a tongue blade. No erosions are present. Which of the following best explains these findings?

1) Diphtheria 2) Leukoplakia - Leukoplakia are flat, white lesions that can range in size and cannot be brushed from the oral mucosa. They warrant referral to a specialist for biopsy as many of these may be or can become cancerous. Most patients with leukoplakia are asymptomatic. This is the most likely diagnosis given this patient's history of chewing tobacco use (chronic irritation) and the clinical presentation. 3) Mononucleosis 4) Oral candidiasis 5) Oral lichen planus

A 68-year-old woman comes to the primary care office because she has had intermittent episodes of dizziness that last between five to 10 minutes during the past several weeks. She says the episodes are triggered by quick head motions. The patient has not had nausea and vomiting, hearing changes, or gait instability. During the physical examination, the Dix-Hallpike maneuver is performed and transient upbeat-torsional nystagmus is noted. Which of the following is the most appropriate next step in management?

1) Epley maneuver - The Epley maneuver is an effective treatment for benign paroxysmal positional vertigo, which is the most likely cause of the dizziness described in the vignette. The pathophysiology of BPPV is of a loose otolith in the semicircular canals causing a false sense of motion. Thus, the premise of the Epley maneuver is to reposition the otolith(s) back to normal and thus eliminate the false sense of motion. 2) Orthostatic blood pressure 3) Referral to a neurologist 4) Restriction of salt and use of diuretics 5) X-ray studies of the head

A 54-year-old man, who had a myocardial infarction last year, comes to his family medicine clinic for treatment of his rhinitis. He complains of congestion as well as a dry cough, and clear rhinorrhea which has lasted for about six weeks. Which of the following medications is the first-line treatment for this patient's diagnosis?

1) Fluticasone - This patient has chronic rhinitis given the time course >4 weeks. Intranasal fluticasone is the treatment of choice in chronic rhinitis. 2) Montelukast 3) Pseudoephedrine 4) Salmeterol 5) Tiotropium

A 4-year-old boy is brought to the clinic because he has had fever and pain in his left ear that are consistent with acute otitis media. This is the patient's fourth episode during the past six months. The mother says the patient has been asymptomatic in between episodes. Which of the following is the most appropriate next step in management?

1) Prescribe amoxicillin prophylaxis 2) Prescribe antihistamine therapy 3) Refer for adenoidectomy 4) Refer for placement of a tympanostomy tube - Recurrent otitis media is defined by three or more episodes of AOM within a six-month period or four or more episodes in a 12-month period. Tympanostomy tube placement has been shown to significantly decrease the number of AOM episodes. 5) Refer for tympanocentesis

A patient presents with loss of the left visual field in each eye. What is the most likely location for the lesion? 1) Frontal lobe 2) Left optic tract 3) Optic chiasm 4) Optic nerve 5) Right optic tract

1) Frontal lobe - The frontal lobe is not involved in vision. This patient has a lesion of the right optic tract. 2) Left optic tract - Lesions in the left optic tract will cause a right homonymous hemianopsia. This patient has a lesion of the right optic tract. 3) Optic chiasm - Lesions of the optic chiasm result in bitemporal hemianopia. This patient has a lesion of the right optic tract. 4) Optic nerve - An optic nerve lesion would cause a complete ipsilateral vision loss (ie: a lesion of the right optic nerve would causes complete right eye blindness). This patient has a lesion of the right optic tract. 5) Right optic tract - This patient has a lesion of the right optic tract. Lesions in the right optic tract will cause a left homonymous hemianopsia.

A 7-year-old boy is brought to your clinic because he was hit in the eye with a ball while playing with his younger brother this morning. He has complained of pain in his eye ever since, and on examination, you notice layered blood in the anterior chamber. Which of the following describes this abnormality? 1) Hyphema 2) Hypopyon 3) Keratitis 4) Pterygium 5) Uveitis

1) Hyphema - Hyphema is the presence blood in the anterior chamber, usually due to blunt trauma or penetrating injury. Patients may also complain of photophobia, pain, nausea, and blurry vision. Patients with hyphema should be seen emergently by an ophthalmologist to exclude open globe injury. 2) Hypopyon - Hypopyon is the presence of pus in the anterior chamber. Patients with hypopyon need to be evaluated urgently by an ophthalmologist, as this is a sign of endophthalmitis, a bacterial infection within the eye, or keratitis. 3) Keratitis - Keratitis, or inflammation of the cornea, can be caused by bacterial, viral, or fungal infections. Prolonged or overnight contact lens use is a major risk factor. Patients will complain of painful eye, foreign body sensation, and difficulty opening the eye. Patients should be evaluated by an ophthalmologist. 4) Pterygium - A pterygium is a vascular triangular mass growing from the nasal canthus toward the cornea. It can extend onto the cornea itself and cause vision impairment, redness, and irritation. It an be removed surgically if causing significant irritation or vision loss. 5) Uveitis - Uveitis is inflammation of the uveal tract, including the iris, ciliary body, and choroid. It can occur due to infection or inflammatory diseases, such as sarcoidosis, inflammatory bowel disease, ankylosing spondylitis, and lupus. Patients should be seen by an ophthalmologist for treatment.

A 19-year-old woman comes to the office because of localized pain and swelling within the floor of the left side of her mouth and jaw for the past 3 days. She states that the pain increases whenever she eats, particularly after eating something that is sour. Physical examination shows a firm mass in the submandibular area with mild to moderate tenderness. Scant pus is expressed from the opening of the submandibular duct when the gland is gently compressed. Which of the following is the most likely diagnosis? 1) Infected branchial cleft cyst 2) Infected thyroglossal duct cyst 3) Parotitis 4) Peritonsillar abscess 5) Sialadenitis

1) Infected branchial cleft cyst - An infected brachial cleft cyst is generally found in the upper neck deep to the sternocleidomastoid muscle. 2) Infected thyroglossal duct cyst - Infected thyroglossal duct cysts are generally in the midline of the neck. 3) Parotitis - Parotitis is infection of the parotid gland which is at the cheek and angle of the mandible. 4) Peritonsillar abscess - A peritonsillar abscess is visualized on the tonsils in the posterior portion of the oropharynx. Patients may also have trismus and airway involvement if the swelling is severe. 5) Sialadenitis - Sialadenitis is an inflammation (often, bacterial infection) of a salivary gland, usually due to an obstructing stone or gland hyposecretion. The most common causative organism is Staphylococcus aureus. Symptoms include swelling, pain, redness, and tenderness. Pus can often be expressed from the duct by compressing the affected gland. Diagnosis is usually clinical.

An 8-year-old girl is brought to the office by her mother because of pain and discomfort in the right ear since waking up in the morning. Otoscopic examination of the ear shows a live insect in the auditory canal. The tympanic membrane is intact. Which of the following interventions is the most appropriate next step in management?

1) Instillation of 10% carbamide peroxide solution followed by insect removal 2) Instillation of 2% lidocaine solution followed by insect removal - The insect should be killed prior to removal, using lidocaine(2%) or mineral oil. The insect may then be removed with use of alligator forceps or curette. Irrigation is the simplest method of foreign body removal provided the tympanic membrane is not perforated. Irrigation with water is contraindicated for soft objects, organic matter or seeds which may swell. 3) Irrigation of the auditory canal with 0.3% gentamicin otic solution 4) Irrigation of the auditory canal with a high pressure pulsating water system 5) Referral to an otolaryngologist in one week for insect removal

A 21-year-old woman who is otherwise healthy comes to the office because of increasing pain in her left ear over the past day since returning from a trip to the Caribbean three days ago. Initially, she had a sense of fullness in the ear, slightly diminished hearing, and mild itching of the ear canal a day before the ear became painful. She denies any trauma to the ear. Temperature is 38.2°C (100.8°F). Palpation of the pinna causes pain. Inspection of the canal shows edema, erythema, and narrowing along with a purulent discharge. There is a small perforation in the tympanic membrane. In addition to the removal of debris from the ear canal, which of the following is the most appropriate treatment?

1) Instillation of acetic acid solution 2) Instillation of dexamethasone and tobramycin solution 3) Instillation of neomycin, polymyxin B, and hydrocortisone solution 4) Instillation of ofloxacin solution - This patient has otitis externa with a ruptured tympanic membrane. Fluoroquinolones do not have any ototoxicity and are the only FDA-approved drug for middle ear use. Ofloxacin otic solution along with ciprofloxacin/dexamethasone are the two topical drops most commonly used. 5) Irrigation of the ear canal - Irrigating the ear when the tympanic membrane is perforated can cause damage to the ossicles and cochlear-vestibular apparatus, resulting in hearing loss, tinnitus, vertigo, and dizziness.

A 24-year-old female patient presents to your clinic complaining of a non-productive cough, nasal congestion and rhinorrhea, and a sore throat for the last four days. Upon physical exam, you find a temperature of 100.8º F (38.2º C), an erythematous oropharynx, and lungs that are clear to auscultation. A rapid streptococcal swab performed in the office is negative. What is the most likely etiology of the patient's suspected diagnosis?

1) Streptococcus pyogenes 2) Adenovirus 3) Epstein-Barr virus 4) Respiratory syncytial virus 5) Rhinovirus With this patient presentation, and negative testing, viral upper respiratory tract infection should be suspected. Rhinovirus is the most common cause of URI about 80% during peak season.

A 22-year-old man comes to the office seeking evaluation for chronic bilateral nasal congestion for the past month. His symptoms began six weeks ago when he developed an upper respiratory infection. At that time, he had a stuffy nose, sore throat, mild cough, and low-grade fever. He treated his symptoms with an over-the-counter multi-symptom cold medicine and a nasal spray containing oxymetazoline. Most of his symptoms resolved after one week, but he has continued to have worsening nasal congestion despite the continued daily use of the nasal spray. Examination of the nose shows clear nasal discharge and bilateral mucosal edema, but no anatomic abnormalities or polyps. The pharynx is normal, and there is no cervical lymphadenopathy. Which of the following is the most appropriate next step in management?

1) Instruct the patient to discontinue use of the nasal spray - This patient has rhinitis medicamentosa caused by overuse of the nasal decongestant spray containing oxymetazoline. This medication should not be used longer than 5 days since prolonged use can decrease its effectiveness, and cause rebound swelling and congestion. Discontinuation of the oxymetazoline is important, and the rebound effect from discontinuation can be managed with the addition of a glucocorticoid nasal spray for short term use. 2) Obtain a Water view x-ray study of the sinuses - Water view x-ray study is sometimes used to evaluate the maxillary sinuses. However, no imaging study is indicated in this scenario. 3) Order a radioallergosorbent test (RAST) for allergies - Allergic rhinosinusitis is a possible underlying cause for chronic nasal congestion, but the patient's time course, signs and symptoms are more suggestive of rhinitis medicamentosa. A RAST study would not be the first step in managing this patient. 4) Prescribe a 10-day course of amoxicillin/clavulanic acid - Antibiotics are not indicated for this condition as there is no suggestion of an infectious cause. 5) Prescribe a daily oral antihistamine - Allergic rhinosinusitis is a possible underlying cause for chronic nasal congestion, but the patient's time course, signs and symptoms are more suggestive of rhinitis medicamentosa.

A 34-year-old man comes to the office because of increasing nasal congestion on the right side of his nose over the past four months. He states that the congestion is positional, and is usually worse when he lays on his left side. Physical examination shows a small, fleshy translucent mass on the inferior turbinate of the right nasal cavity. The remainder of the examination shows no abnormalities. Which of the following is the most appropriate initial management of this patient's condition?

1) Intranasal application of corticosteroid spray - Topical corticosteroids are beneficial in the treatment of nasal polyps. They are effective in reducing symptoms, nasal obstruction, and polyp size. Additionally, they help in preventing polyp regrowth following surgery. Administration of oral glucocorticoids are also useful, particularly with larger polyps. Surgical excision may also be indicated with large polyps. 2) Intranasal application of cromolyn sodium spray 3) Intranasal application of decongestant spray 4) Intravenous administration of amphotericin B 5) Oral administration of antihistamine

A patient presents with visual field loss of the lateral portion of each eye. What is the most likely location for the lesion?

1) Left optic tract - A left optic tract lesion would cause right homonymous hemianopsia. 2) Optic chiasm - Bitemporal hemianopsia is caused by a lesion at the optic chiasm. 3) Optic nerve - An optic nerve lesion would cause a complete ipsilateral vision loss (a lesion of the right optic nerve would causes complete right eye blindness). 4) Optic radiation - A lesion at the optic radiation would cause quadrantanopsia, a loss of a quarter of the field of vision. 5) Right optic tract - A right optic tract lesion would cause left homonymous hemianopsia.

A 32-year-old female presents complaining of chronically itchy, burning, and dry eyes. She admits that they feel very tired in the afternoon and evening. She has tried artificial tear drops without significant relief. What physical exam findings would support your suspected diagnosis?

1) Lisch nodules found on the iris - This would support a diagnosis of neurofibromatosis, not dry eye. 2) Poor tear film with punctate epithelial erosions - This is a physical exam finding consistent with the suspected diagnosis of chronic dry eye. 3) Subconjunctival hemorrhage - Subconjunctival hemorrhage is a benign condition in which blood is seen in the conjunctiva. Reassurance is the only treatment necessary, and SCH usually resolves within 2 weeks. 4) Unequal pupil size - This is anisocoria. 5) Unequal pupillary light reflexion - This is a finding in strabismus.

A 27-year-old woman comes to your office with upper respiratory infection symptoms. On otoscopic exam of the left ear, you visualize straw-colored fluid behind the tympanic membrane and prominent landmarks. The right ear appears normal. You suggest conservative management of her URI. At a follow-up visit 3 months later, she states her URI symptoms have resolved but complains of some hearing loss in the left ear. Her otoscopic findings are unchanged. What is the next best step in the management of this patient?

1) No further evaluation is necessary 2) Order a CT scan 3) Perform influenza testing 4) Prescribe amoxicillin 5) Refer to ENT - The otoscopic findings in this patient suggest serous otitis media. This condition is less common in adults, and typically occurs following an URI, chronic allergic rhinitis, or barotrauma. Adults with persistent unilateral serous otitis media longer than 12 weeks should be referred to ENT for further management with possible myringotomy +/- tube placement.

A 34-year-old man comes to your family medicine clinic with pain in the inside of his mouth and a dry mouth for the past three days. On physical examination, you note an area of swelling and discrete round, hard sub-centimeter induration on the floor of the mouth near the frenulum of the tongue. Which of the following best explains this patient's symptoms and physical exam findings? 1) Oral candidiasis 2) Oropharyngeal carcinoma 3) Parotitis 4) Sialadenitis 5) Streptococcal infection

1) Oral candidiasis - Oral candidiasis, or thrush, is characterized by white plaques on the tongue, buccal mucosa, or palate, that are easily scraped away, with underlying erythema. Remember, 'thrush will brush'. It occurs in immunocompromised patients, patients using corticosteroid inhalers, patients who use dentures, and in those recently treated with antibiotics. Treatment involves topical antifungal agents. 2) Oropharyngeal carcinoma - Oropharyngeal carcinoma presents as persistent, non-healing oral lesions (papules, plaques, ulcers) that may be associated with pain. Squamous cell carcinoma is common, especially in patients that smoke or use chewing tobacco. 3) Parotitis - Parotitis is a relatively rare condition that is commonly associated with mumps, anorexia nervosa, dehydration, and oral cancers. Patients will have preauricular swelling, tenderness, trismus, and fever. Toxic-appearing patients need to be treated with IV antibiotics. Patients can also be told to apply warm compresses, massage the area, and suck on sour candy. 4) Sialadenitis - Sialadenitis is inflammation of a salivary gland secondary either to obstructive or suppurative processes. 80-90% of salivary stones arise from the submandibular gland at the os of Wharton's duct, which is located on the floor of the mouth near the frenulum of the tongue. Treatment is conservative. Patients can also be told to apply warm compresses, massage the area, and suck on sour candy. NSAIDs can be used for pain. Suspected bacterial sialadenitis should be treated with antibiotics. 5) Streptococcal infection - Patients with streptococcal infection will complain of sudden onset of sore throat, odynophagia, and fever. The Centor Criteria can be used in the evaluation of streptococcal pharyngitis, with presence at least 3 of the following findings warranting diagnostic testing: fever, tonsillar exudates, absence of cough, and tender anterior cervical lymphadenopathy. Treatment involves antibiotics.

A 6-year-old girl is brought to the emergency department by her parents because she has had severe swelling of her right cheek for the past day. She was evaluated by her pediatrician three days ago because of fever, headache, and anorexia, and a nonspecific viral illness was diagnosed. Physical examination shows a very tender right parotid gland with swelling severe enough to obscure the angle of the mandible. Which of the following viral illnesses is most commonly associated with this disorder?

A. Chickenpox B. Fifth disease C. Hand, foot, and mouth disease D. Measles E. Mumps The mumps virus, a paramyxovirus, is the most common viral cause of parotitis. Initial symptoms include fever, headache, myalgia, and/or anorexia. Parotitis (either unilateral or bilateral) generally develops shortly afterwards.

A 34-year-old man is seen in the office for consultation one day after a spontaneous rupture of the tympanic membrane. While camping two days ago, he developed acute otitis media of the right ear. Yesterday, he had spontaneous relief of ear pain along with some bloody purulent discharge from the right ear canal. He has no subjective loss of hearing. Physical examination of the ear shows residual dried blood in the auditory canal, and a clot overlying a small perforation site in the pars tensa of the tympanic membrane. Which of the following is the most appropriate advice to give this patient regarding his condition?

A. Arrange for transportation to the emergency department for urgent evaluation B. Daily insertion of an ear wick soaked with aminoglycoside otic solution for one week C. Irrigate the ear canal daily in a shower to remove residual debris D. Keep the ear canal dry to allow the perforation to heal spontaneously The auditory canal should be kept dry until the tympanic membrane perforation heals. E. Schedule an appointment with an otolaryngologist for tympanostomy tube placement

A 9-year-old boy is brought to the emergency department because he has had swelling and redness of his left eye as well as blurry vision in the eye since he was hit in the head during a soccer game two days ago. The patient rates the pain around his eye as a 7 on a 10-point scale. Temperature is 38.3°C (101.0°F), pulse rate is 120/min, respirations are 20/min, and blood pressure is 120/70 mmHg. Physical examination shows proptosis and pain with extraocular movements. Which of the following is the most appropriate diagnostic study for this patient's condition?

A. CT scan with contrast CT scan with contrast is the study of choice in suspected orbital cellulitis and provides diagnostic information regarding how extensive the infection is and differentiates preseptal (periorbital cellulitis) from postseptal (orbital cellulitis). B. CT scan without contrast C. Orbital ultrasound D. Sinus aspiration and culture E. Water's view x-ray

A 12-year-old boy is seen in the office because of a sore throat for the past two days. He says it hurts to swallow, and he feels achy all over. Physical examination of the oropharynx shows bilaterally enlarged and erythematous tonsils covered with patchy exudate. Rapid antigen detection testing for group A Streptococcus is positive. Which of the following antibiotics is the most appropriate initial treatment for this patient's condition?

A. Chloramphenicol B. Ciprofloxacin C. Penicillin Oral penicillin remains the drug of choice for treating streptococcal tonsillitis/pharyngitis. D. Tetracycline E. Trimethoprim-sulfamethoxazole

A 77-year-old patient at the nursing home is complaining of decreased ability to hear from their right ear. On physical examination, you note that the sound from the Weber test is louder on the right, and that the Rinne test of the right ear reveals bone conduction is greater than air conduction. Which of the following is the most likely cause of this patient's hearing loss?

A. Acoustic neuroma of the left ear B. Acoustic neuroma of the right ear C. Cerumen impaction of the left ear D. Cerumen impaction of the right ear Cerumen impaction is the most common cause of conductive hearing loss in the elderly. Other causes of conductive hearing loss include trauma, foreign body, otitis media, otosclerosis, and tympanic membrane perforation. Generally with conductive hearing loss Weber will lateralize to the affected ear (in this case, the right ear), and Rinne will reveal bone conduction > air conduction. E. Meniere disease

A 55-year-old patient presents with painless vision loss of the right eye for the last 3 hours. They state, "I can't see the top portion of things out of my right eye." They also admit to seeing "floaters." On visual inspection, the eye looks normal. You prepare to conduct a fundoscopic examination. Which of the following most likely explains this patient's complaint?

A. Acute angle-closure glaucoma B. Corneal abrasion C. Hyphema D. Retinal artery occlusion E. Retinal detachment This patient likely has a retinal detachment. Retinal detachments cause a fairly acute, painless loss of portions of vision and patients often complain of seeing floaters.

A 32-year-old man comes to the urgent care clinic because of a severe sore throat for the past two days. He states that the pain is made worse with swallowing, and as he speaks, he speaks with a "hot potato" voice. Physical examination shows erythema of the soft palate and tonsils. The left tonsil is markedly enlarged, and the uvula is displaced away from the midline toward the right. Which of the following is the most likely explanation for this patient's symptoms and physical exam findings?

A. Acute necrotizing ulcerative gingivitis B. Diphtheria C. Epiglottitis D. Laryngitis E. Peritonsillar abscess Peritonsillar abscess is a collection of pus that lies between the tonsillar capsule, and superior pharyngeal constrictor muscle. It is generally thought to result as a complication of tonsillitis. Consequently, it has a similar clinical presentation, and infective etiology. A progressively severe sore throat on one side, and pain during swallowing are usually the earliest symptoms. As the abscess develops, persistent pain in the peritonsillar area, fever, headache, and a distortion of vowels ("hot potato voice") may appear. Physical signs include redness and swelling in the tonsillar area of the affected side, and swelling of the jugulodigastric lymph nodes. The uvula may be displaced towards the unaffected side.

A 32-year-old man with diabetes comes to the emergency department with two days of intense, deep ear pain and purulent drainage from the left ear. Physical examination reveals an erythematous and edematous canal which contains some debris and granulation tissue. The tympanic membrane is barely visible but appears intact. The surrounding external ear is also indurated and erythematous. The patient has a fever of 102.1F. Which of the following is the next best step in the care of this patient?

A. Admit the patient for hyperbaric oxygen therapy. B. Admit the patient to the hospital for intravenous antibiotics. In diabetic or immunocompromised patients, the risk of malignant otitis externa (osteomyelitis of the skull base) is high, and treatment should begin immediately with IV anti-pseudomonal antibiotics. Ciprofloxacin is the first-line antibiotic. A CT may also be done to evaluate for bone erosion. C. Prescribe ciprofloxacin/dexamethasone ear drops, and counsel patient to keep the ear canal dry for at least a week. D. Prescribe oral amoxicillin, and schedule a follow up visit for two weeks to make sure infection has cleared. E. Refer the patient to an otolaryngologist in one week.

A 26-year-old man comes to the clinic because he has had recurrent small, painful mouth ulcers that come and go during the past four months. He says he currently has one on the inside of his lower lip. His medical history includes no chronic disease conditions or surgeries, and he takes no medications. He drinks seven to 10 alcoholic beverages per week, is a social cigarette smoker, and does not use illicit drugs. He has been sexually active with two different partners during the past year and has not been tested for sexually transmitted infections. Temperature is 37.1°C (98.8°F), pulse rate is 80/min, respirations are 16/min, and blood pressure is 128/87 mmHg. Oxygen saturation is 98% on room air. A photograph of his lip is shown. Which of the following laboratory studies is most likely to determine the diagnosis?

A. Antinuclear antibody test B. Rapid plasma reagin test with reflex to Treponema pallidum antibodies C. Skin biopsy with dermatopathology D. Tzanck test A Tzanck test can help rule out herpes simplex virus, which is the most likely alternative to the diagnosis of an oral aphthous ulcer, which is the correct diagnosis. E. White blood cell count with differential

A 17-year-old male is brought to the emergency department because of acute onset of right eye pain, decreased vision from the right eye, and mucopurulent discharge. While eliciting the history you discover that he began wearing contact lenses a few months earlier, and does not remember the last time he removed and cleaned them. When examining the eye with fluorescein, you note uptake in a dendritic pattern around the cornea. Which of the following is the most likely diagnosis?

A. Corneal abrasion B. Episcleritis C. Herpes keratitis Herpes keratitis is often due to contact lens use and classically is seen by fluorescein uptake in a dendritic pattern. This can be a very serious infection, and ophthalmology should be consulted immediately. D. Ocular foreign body E. Uveitis

A 5-year-old girl is brought to your office by her mother who has noticed a rash on the palms of her hands and soles of her feet for the past 3 days. She also reports that her daughter has had a fever of 100.5º F (38.1º C) and significantly decreased appetite. Which of the following physical examination findings would you suspect in this patient?

A. Decreased vibratory sensation B. Diffuse abdominal tenderness with palpable spleen tip C. Painful, small gray lesions on an erythematous base seen on the oral mucosa This finding describes the aphthous ulcers commonly seen in hand, foot and mouth disease caused by Coxsackievirus A16. These patients typically have painful oral ulcers, maculopapular rash of hands and feet, fever, and refusal to eat. It is most common in kids younger than 7 years of age. It is a self-limiting disease that usually resolves in a week. D. Tender anterior cervical lymphadenopathy E. White plaques on the buccal mucosa that remain intact when brushed with the tongue blade

A 7-year-old girl who is otherwise healthy is brought to the urgent care by her mother because she has had fever, sore throat, and decreased appetite for the past two days. She is up to date with her immunizations. Temperature is 103.2ºF (39.6ºC). On physical examination, her pharynx has a whitish exudate in patches over the tonsils, and she has tender cervical lymphadenopathy. You also note palatal petechiae and an absence of a cough. Which of the following is the most likely diagnosis? A. Diphtheria B. Epiglottitis C. Mononucleosis D. Streptococcal pharyngitis E. Viral upper respiratory infection

A. Diphtheria B. Epiglottitis C. Mononucleosis D. Streptococcal pharyngitis Strep pharyngitis is the most likely diagnosis. This patient meets Centor criteria, which include: fever, exudative tonsillitis, tender cervical lymphadenopathy, and absence of a cough. Palatal petechiae may also be present. E. Viral upper respiratory infection


Related study sets

Missed questions from all chapters for exam 3

View Set

ATI Testbank Questions- OB Exam #2 part I

View Set

Chapter 56 PrepU: Dermatological

View Set

Chapter 4. Activity-Based Costing

View Set